LSAT and Law School Admissions Forum

Get expert LSAT preparation and law school admissions advice from PowerScore Test Preparation.

 Administrator
PowerScore Staff
  • PowerScore Staff
  • Posts: 8916
  • Joined: Feb 02, 2011
|
#38948
Complete Question Explanation
(The complete setup for this game can be found here: lsat/viewtopic.php?t=15134)

The correct answer choice is (A)

This question asks us to identify a variable that cannot be third. Even without the use of Not-Laws, our two sequencing chains can quickly and efficiently help us identify the correct answer choice.

Answer choice (A): This is the correct answer choice. In Template 1, G must perform after at least three other variables—P, S, and K. In Template 2, G must perform the last solo. Clearly, there is no solution where G performs third.

Answer choice (B): This answer choice is incorrect, because K could perform third, after P and V, in Template 1.

Answer choice (C): This answer choice is incorrect, because S could perform third in each of the two templates.

Answer choice (D): This answer choice is incorrect, because T could perform third in Template 1:

PT72 - Game_#1_#5_diagram 1.png


Answer choice (E): This answer choice is incorrect, because V could perform third in each of the two templates.
You do not have the required permissions to view the files attached to this post.

Get the most out of your LSAT Prep Plus subscription.

Analyze and track your performance with our Testing and Analytics Package.